matheraum.de
Raum für Mathematik
Offene Informations- und Nachhilfegemeinschaft

Für Schüler, Studenten, Lehrer, Mathematik-Interessierte.
Hallo Gast!einloggen | registrieren ]
Startseite · Forum · Wissen · Kurse · Mitglieder · Team · Impressum
Forenbaum
^ Forenbaum
Status Mathe
  Status Schulmathe
    Status Primarstufe
    Status Mathe Klassen 5-7
    Status Mathe Klassen 8-10
    Status Oberstufenmathe
    Status Mathe-Wettbewerbe
    Status Sonstiges
  Status Hochschulmathe
    Status Uni-Analysis
    Status Uni-Lin. Algebra
    Status Algebra+Zahlentheo.
    Status Diskrete Mathematik
    Status Fachdidaktik
    Status Finanz+Versicherung
    Status Logik+Mengenlehre
    Status Numerik
    Status Uni-Stochastik
    Status Topologie+Geometrie
    Status Uni-Sonstiges
  Status Mathe-Vorkurse
    Status Organisatorisches
    Status Schule
    Status Universität
  Status Mathe-Software
    Status Derive
    Status DynaGeo
    Status FunkyPlot
    Status GeoGebra
    Status LaTeX
    Status Maple
    Status MathCad
    Status Mathematica
    Status Matlab
    Status Maxima
    Status MuPad
    Status Taschenrechner

Gezeigt werden alle Foren bis zur Tiefe 2

Navigation
 Startseite...
 Neuerdings beta neu
 Forum...
 vorwissen...
 vorkurse...
 Werkzeuge...
 Nachhilfevermittlung beta...
 Online-Spiele beta
 Suchen
 Verein...
 Impressum
Das Projekt
Server und Internetanbindung werden durch Spenden finanziert.
Organisiert wird das Projekt von unserem Koordinatorenteam.
Hunderte Mitglieder helfen ehrenamtlich in unseren moderierten Foren.
Anbieter der Seite ist der gemeinnützige Verein "Vorhilfe.de e.V.".
Partnerseiten
Dt. Schulen im Ausland: Mathe-Seiten:Weitere Fächer:

Open Source FunktionenplotterFunkyPlot: Kostenloser und quelloffener Funktionenplotter für Linux und andere Betriebssysteme
StartseiteMatheForenHochschulPhysikIntegration v. Arbeitsbeträgen
Foren für weitere Schulfächer findest Du auf www.vorhilfe.de z.B. Philosophie • Religion • Kunst • Musik • Sport • Pädagogik
Forum "HochschulPhysik" - Integration v. Arbeitsbeträgen
Integration v. Arbeitsbeträgen < HochschulPhysik < Physik < Naturwiss. < Vorhilfe
Ansicht: [ geschachtelt ] | ^ Forum "HochschulPhysik"  | ^^ Alle Foren  | ^ Forenbaum  | Materialien

Integration v. Arbeitsbeträgen: Verständnisfrage
Status: (Frage) beantwortet Status 
Datum: 09:10 Fr 06.11.2015
Autor: marc518205

Aufgabe
Integration der infinitesimalen Arbeitsbeträge dA entlang des Wegs zwischen zwei Punkten [mm] P_{0} [/mm] = [mm] P_{\vec{s0}} [/mm] und [mm] P_{1} [/mm] = [mm] P_{\vec{s1}}: [/mm]

A = [mm] \integral_{\vec{s0}}^{\vec{s1}}{\vec{F}d{\vec{s}} = \integral_{P0}^{P1}{(F_{x}}dx+F_{y}dy+F_{z}dz)} [/mm]

NG2 (2. Newtonsches Gesetz) und Übergang zur Parameterdarstellung [mm] \vec{s}(t): [/mm]

hallo und danke schon mal für eure hilfe. soweit sind die angaben für mich noch irgendwie nachvollziehbar. bei den nächsten schritten blick ich dann nicht mehr durch. könnt ihr mir da bitte weiterhelfen?

A =  [mm] \integral_{\vec{s0}}^{\vec{s1}}{\vec{F}d\vec{s}} [/mm]
(ich kann an stelle von [mm] \vec{F} [/mm] auch [mm] \vec{p}' [/mm] (ableitung von p) schreiben oder? und formal erweitere ich durch dt.)

= [mm] \integral_{t0}^{t1}{\vec{p}' \bruch{d\vec{s}}{dt} dt} [/mm]
(hier ändert sich die integration vom weg richtung zeit, also von s zu t und darum brauch ich die geschwindigkteit  im integral, gegeben durch [mm] \bruch{d\vec{s}}{dt} [/mm] = [mm] \bruch{p}{m} [/mm] = v. bin ich soweit noch richtig?)

Es geht dann noch weiter, aber mich würde interessieren ob ich soweit noch richtig bin. danke nochmals für eure hilfe.


        
Bezug
Integration v. Arbeitsbeträgen: Antwort
Status: (Antwort) fertig Status 
Datum: 09:40 Fr 06.11.2015
Autor: hippias


> Integration der infinitesimalen Arbeitsbeträge dA entlang
> des Wegs zwischen zwei Punkten [mm]P_{0}[/mm] = [mm]P_{\vec{s0}}[/mm] und
> [mm]P_{1}[/mm] = [mm]P_{\vec{s1}}:[/mm]
>  
> A = [mm]\integral_{\vec{s0}}^{\vec{s1}}{\vec{F}d{\vec{s}} = \integral_{P0}^{P1}{(F_{x}}dx+F_{y}dy+F_{z}dz)}[/mm]
>  
> NG2 (2. Newtonsches Gesetz) und Übergang zur
> Parameterdarstellung [mm]\vec{s}(t):[/mm]
>  hallo und danke schon mal für eure hilfe. soweit sind die
> angaben für mich noch irgendwie nachvollziehbar. bei den
> nächsten schritten blick ich dann nicht mehr durch. könnt
> ihr mir da bitte weiterhelfen?
>  
> A =  [mm]\integral_{\vec{s0}}^{\vec{s1}}{\vec{F}d\vec{s}}[/mm]
> (ich kann an stelle von [mm]\vec{F}[/mm] auch [mm]\vec{p}'[/mm] (ableitung
> von p) schreiben oder?

Ja, Kraft ist die Ableitung des Impulses nach der Zeit; ich wuerde daher aber lieber [mm] $\dot{\vec{p}}$ [/mm] schreiben.

> und formal erweitere ich durch dt.)
>  
> = [mm]\integral_{t0}^{t1}{\vec{p}' \bruch{d\vec{s}}{dt} dt}[/mm]
>  
> (hier ändert sich die integration vom weg richtung zeit,
> also von s zu t und darum brauch ich die geschwindigkteit  
> im integral, gegeben durch [mm]\bruch{d\vec{s}}{dt}[/mm] =
> [mm]\bruch{p}{m}[/mm] = v. bin ich soweit noch richtig?)

Ja. Du drueckst Dich aber nicht sonderlich gut aus: Du hast beim Integral eine Koordinatentransformation vorgenommen oder eine Umparametrisierung vorgenommen o.s.ä. Die Gleichung ergibt sich aus der Substitutionsregel der Integration.

>  
> Es geht dann noch weiter, aber mich würde interessieren ob
> ich soweit noch richtig bin. danke nochmals für eure
> hilfe.
>  

Sieht für mich bisher nicht falsch aus.

Bezug
                
Bezug
Integration v. Arbeitsbeträgen: Frage (beantwortet)
Status: (Frage) beantwortet Status 
Datum: 11:22 Fr 06.11.2015
Autor: marc518205


> > Integration der infinitesimalen Arbeitsbeträge dA entlang
> > des Wegs zwischen zwei Punkten [mm]P_{0}[/mm] = [mm]P_{\vec{s0}}[/mm] und
> > [mm]P_{1}[/mm] = [mm]P_{\vec{s1}}:[/mm]
>  >  
> > A = [mm]\integral_{\vec{s0}}^{\vec{s1}}{\vec{F}d{\vec{s}} = \integral_{P0}^{P1}{(F_{x}}dx+F_{y}dy+F_{z}dz)}[/mm]
>  
> >  

> > NG2 (2. Newtonsches Gesetz) und Übergang zur
> > Parameterdarstellung [mm]\vec{s}(t):[/mm]
>  >  hallo und danke schon mal für eure hilfe. soweit sind
> die
> > angaben für mich noch irgendwie nachvollziehbar. bei den
> > nächsten schritten blick ich dann nicht mehr durch. könnt
> > ihr mir da bitte weiterhelfen?
>  >  
> > A =  [mm]\integral_{\vec{s0}}^{\vec{s1}}{\vec{F}d\vec{s}}[/mm]
> > (ich kann an stelle von [mm]\vec{F}[/mm] auch [mm]\vec{p}'[/mm] (ableitung
> > von p) schreiben oder?
> Ja, Kraft ist die Ableitung des Impulses nach der Zeit; ich
> wuerde daher aber lieber [mm]\dot{\vec{p}}[/mm] schreiben.
>  
> > und formal erweitere ich durch dt.)
>  >  
> > = [mm]\integral_{t0}^{t1}{\vec{p}' \bruch{d\vec{s}}{dt} dt}[/mm]
>  
> >  

> > (hier ändert sich die integration vom weg richtung zeit,
> > also von s zu t und darum brauch ich die geschwindigkteit  
> > im integral, gegeben durch [mm]\bruch{d\vec{s}}{dt}[/mm] =
> > [mm]\bruch{p}{m}[/mm] = v. bin ich soweit noch richtig?)
>  Ja. Du drueckst Dich aber nicht sonderlich gut aus: Du
> hast beim Integral eine Koordinatentransformation
> vorgenommen oder eine Umparametrisierung vorgenommen
> o.s.ä. Die Gleichung ergibt sich aus der
> Substitutionsregel der Integration.
>
> >  

> > Es geht dann noch weiter, aber mich würde interessieren ob
> > ich soweit noch richtig bin. danke nochmals für eure
> > hilfe.
>  >  
> Sieht für mich bisher nicht falsch aus.


ok, danke. dann gehts weiter und hier blick ich nicht mehr durch:

= [mm] \integral_{t0}^{t1}{\dot{\vec{p}} \bruch{d\vec{s}}{dt} dt} [/mm]

= [mm] \integral_{t0}^{t1}{\dot{\vec{p}} \bruch{\vec{p}}{m} dt} [/mm]
hier wird [mm] \bruch{\vec{p}}{m} [/mm] statt [mm] \bruch{d\vec{s}}{dt} [/mm] eingesetzt. Aber wie komme ich nun zum nächsten integral, genau diesen schritt kann ich nicht nachvollziehen:

=  [mm] \integral_{t0}^{t1}{\bruch{1}{2m} \bruch{d\vec{p}^2}{dt} dt} [/mm]

in der vorlesung wurde erklärt, dass es die produktregel ist. dann würde ich das mal so schreiben:
[mm] \integral_{t0}^{t1}{\bruch{d\vec{p}}{dt} \bruch{\vec{p}}{m} dt} [/mm]
Kann ich das so machen und jetzt so weiter vorgehen?:
  [mm] \integral_{t0}^{t1}{\bruch{1}{m} \bruch{d\vec{p}^2}{dt} dt} [/mm]
und [mm] \bruch{d\vec{p}^2}{dt} [/mm] ergit [mm] 2(\vec{p}+\dot{\vec{p}}) [/mm]
jetzt weiß ich nicht mehr weiter... es wird zwar durch 2 dividiert aber warum, ich versteh das nicht... oder hab ich weiter oben schon einen fehler? danke nochmals


Bezug
                        
Bezug
Integration v. Arbeitsbeträgen: Antwort
Status: (Antwort) fertig Status 
Datum: 12:30 Fr 06.11.2015
Autor: notinX

Hallo,

> > > Integration der infinitesimalen Arbeitsbeträge dA entlang
> > > des Wegs zwischen zwei Punkten [mm]P_{0}[/mm] = [mm]P_{\vec{s0}}[/mm] und
> > > [mm]P_{1}[/mm] = [mm]P_{\vec{s1}}:[/mm]
>  >  >  
> > > A = [mm]\integral_{\vec{s0}}^{\vec{s1}}{\vec{F}d{\vec{s}} = \integral_{P0}^{P1}{(F_{x}}dx+F_{y}dy+F_{z}dz)}[/mm]
>  
> >  

> > >  

> > > NG2 (2. Newtonsches Gesetz) und Übergang zur
> > > Parameterdarstellung [mm]\vec{s}(t):[/mm]
>  >  >  hallo und danke schon mal für eure hilfe. soweit
> sind
> > die
> > > angaben für mich noch irgendwie nachvollziehbar. bei den
> > > nächsten schritten blick ich dann nicht mehr durch. könnt
> > > ihr mir da bitte weiterhelfen?
>  >  >  
> > > A =  [mm]\integral_{\vec{s0}}^{\vec{s1}}{\vec{F}d\vec{s}}[/mm]
> > > (ich kann an stelle von [mm]\vec{F}[/mm] auch [mm]\vec{p}'[/mm] (ableitung
> > > von p) schreiben oder?
> > Ja, Kraft ist die Ableitung des Impulses nach der Zeit; ich
> > wuerde daher aber lieber [mm]\dot{\vec{p}}[/mm] schreiben.
>  >  
> > > und formal erweitere ich durch dt.)
>  >  >  
> > > = [mm]\integral_{t0}^{t1}{\vec{p}' \bruch{d\vec{s}}{dt} dt}[/mm]
>  
> >  

> > >  

> > > (hier ändert sich die integration vom weg richtung zeit,
> > > also von s zu t und darum brauch ich die geschwindigkteit  
> > > im integral, gegeben durch [mm]\bruch{d\vec{s}}{dt}[/mm] =
> > > [mm]\bruch{p}{m}[/mm] = v. bin ich soweit noch richtig?)
>  >  Ja. Du drueckst Dich aber nicht sonderlich gut aus: Du
> > hast beim Integral eine Koordinatentransformation
> > vorgenommen oder eine Umparametrisierung vorgenommen
> > o.s.ä. Die Gleichung ergibt sich aus der
> > Substitutionsregel der Integration.
> >
> > >  

> > > Es geht dann noch weiter, aber mich würde interessieren ob
> > > ich soweit noch richtig bin. danke nochmals für eure
> > > hilfe.
>  >  >  
> > Sieht für mich bisher nicht falsch aus.
>
>
> ok, danke. dann gehts weiter und hier blick ich nicht mehr
> durch:
>  
> = [mm]\integral_{t0}^{t1}{\dot{\vec{p}} \bruch{d\vec{s}}{dt} dt}[/mm]
>  
> = [mm]\integral_{t0}^{t1}{\dot{\vec{p}} \bruch{\vec{p}}{m} dt}[/mm]
>  
> hier wird [mm]\bruch{\vec{p}}{m}[/mm] statt [mm]\bruch{d\vec{s}}{dt}[/mm]
> eingesetzt. Aber wie komme ich nun zum nächsten integral,
> genau diesen schritt kann ich nicht nachvollziehen:
>  
> =  [mm]\integral_{t0}^{t1}{\bruch{1}{2m} \bruch{d\vec{p}^2}{dt} dt}[/mm]

das ist einfach nur eine Umformung des Terms. Berechne doch mal die Ableitung: [mm] $\frac{1}{2m}\frac{\mathrm{d}\vec{p}^{2}}{\mathrm{d}t}=?$ [/mm]

>
> in der vorlesung wurde erklärt, dass es die produktregel
> ist. dann würde ich das mal so schreiben:

Ich würde eher die Kettenregel nehmen, aber im Prinzip ist das egal.

>   [mm]\integral_{t0}^{t1}{\bruch{d\vec{p}}{dt} \bruch{\vec{p}}{m} dt}[/mm]

Diese Schreibweise ist nicht eindeutig, da man nicht genau weiß auf welche Faktoren sich die Ableitung bezieht. Besser so:
[mm] $\frac{1}{m}\frac{\mathrm{d}}{\mathrm{d}t}(\vec{p}\cdot\vec{p})$ [/mm]
Wenn Du davon jetzt die Ableitung berechnest und mit dem Ausgangsterm vergleichst, sollte auch klar sein warum durch 2 dividiert wird.

>  
> Kann ich das so machen und jetzt so weiter vorgehen?:
>    [mm]\integral_{t0}^{t1}{\bruch{1}{m} \bruch{d\vec{p}^2}{dt} dt}[/mm]
>  
> und [mm]\bruch{d\vec{p}^2}{dt}[/mm] ergit [mm]2(\vec{p}+\dot{\vec{p}})[/mm]

[notok]
Du solltest besser nochmal die Ableitungsregeln wiederholen ;-)

>  jetzt weiß ich nicht mehr weiter... es wird zwar durch 2
> dividiert aber warum, ich versteh das nicht... oder hab ich
> weiter oben schon einen fehler? danke nochmals
>  

Gruß,

notinX

Bezug
                                
Bezug
Integration v. Arbeitsbeträgen: Frage (beantwortet)
Status: (Frage) beantwortet Status 
Datum: 13:20 Fr 06.11.2015
Autor: marc518205


> Hallo,
>  
> > > > Integration der infinitesimalen Arbeitsbeträge dA entlang
> > > > des Wegs zwischen zwei Punkten [mm]P_{0}[/mm] = [mm]P_{\vec{s0}}[/mm] und
> > > > [mm]P_{1}[/mm] = [mm]P_{\vec{s1}}:[/mm]
>  >  >  >  
> > > > A = [mm]\integral_{\vec{s0}}^{\vec{s1}}{\vec{F}d{\vec{s}} = \integral_{P0}^{P1}{(F_{x}}dx+F_{y}dy+F_{z}dz)}[/mm]
>  
> >  

> > >  

> > > >  

> > > > NG2 (2. Newtonsches Gesetz) und Übergang zur
> > > > Parameterdarstellung [mm]\vec{s}(t):[/mm]
>  >  >  >  hallo und danke schon mal für eure hilfe. soweit
> > sind
> > > die
> > > > angaben für mich noch irgendwie nachvollziehbar. bei den
> > > > nächsten schritten blick ich dann nicht mehr durch. könnt
> > > > ihr mir da bitte weiterhelfen?
>  >  >  >  
> > > > A =  [mm]\integral_{\vec{s0}}^{\vec{s1}}{\vec{F}d\vec{s}}[/mm]
> > > > (ich kann an stelle von [mm]\vec{F}[/mm] auch [mm]\vec{p}'[/mm] (ableitung
> > > > von p) schreiben oder?
> > > Ja, Kraft ist die Ableitung des Impulses nach der Zeit; ich
> > > wuerde daher aber lieber [mm]\dot{\vec{p}}[/mm] schreiben.
>  >  >  
> > > > und formal erweitere ich durch dt.)
>  >  >  >  
> > > > = [mm]\integral_{t0}^{t1}{\vec{p}' \bruch{d\vec{s}}{dt} dt}[/mm]
>  
> >  

> > >  

> > > >  

> > > > (hier ändert sich die integration vom weg richtung zeit,
> > > > also von s zu t und darum brauch ich die geschwindigkteit  
> > > > im integral, gegeben durch [mm]\bruch{d\vec{s}}{dt}[/mm] =
> > > > [mm]\bruch{p}{m}[/mm] = v. bin ich soweit noch richtig?)
>  >  >  Ja. Du drueckst Dich aber nicht sonderlich gut aus:
> Du
> > > hast beim Integral eine Koordinatentransformation
> > > vorgenommen oder eine Umparametrisierung vorgenommen
> > > o.s.ä. Die Gleichung ergibt sich aus der
> > > Substitutionsregel der Integration.
> > >
> > > >  

> > > > Es geht dann noch weiter, aber mich würde interessieren ob
> > > > ich soweit noch richtig bin. danke nochmals für eure
> > > > hilfe.
>  >  >  >  
> > > Sieht für mich bisher nicht falsch aus.
> >
> >
> > ok, danke. dann gehts weiter und hier blick ich nicht mehr
> > durch:
>  >  
> > = [mm]\integral_{t0}^{t1}{\dot{\vec{p}} \bruch{d\vec{s}}{dt} dt}[/mm]
>  
> >  

> > = [mm]\integral_{t0}^{t1}{\dot{\vec{p}} \bruch{\vec{p}}{m} dt}[/mm]
>  
> >  

> > hier wird [mm]\bruch{\vec{p}}{m}[/mm] statt [mm]\bruch{d\vec{s}}{dt}[/mm]
> > eingesetzt. Aber wie komme ich nun zum nächsten integral,
> > genau diesen schritt kann ich nicht nachvollziehen:
>  >  
> > =  [mm]\integral_{t0}^{t1}{\bruch{1}{2m} \bruch{d\vec{p}^2}{dt} dt}[/mm]
>
> das ist einfach nur eine Umformung des Terms. Berechne doch
> mal die Ableitung:
> [mm]\frac{1}{2m}\frac{\mathrm{d}\vec{p}^{2}}{\mathrm{d}t}=?[/mm]
>  
> >
> > in der vorlesung wurde erklärt, dass es die produktregel
> > ist. dann würde ich das mal so schreiben:
>  
> Ich würde eher die Kettenregel nehmen, aber im Prinzip ist
> das egal.
>  
> >   [mm]\integral_{t0}^{t1}{\bruch{d\vec{p}}{dt} \bruch{\vec{p}}{m} dt}[/mm]

>  
> Diese Schreibweise ist nicht eindeutig, da man nicht genau
> weiß auf welche Faktoren sich die Ableitung bezieht.
> Besser so:
>  
> [mm]\frac{1}{m}\frac{\mathrm{d}}{\mathrm{d}t}(\vec{p}\cdot\vec{p})[/mm]
>  Wenn Du davon jetzt die Ableitung berechnest und mit dem
> Ausgangsterm vergleichst, sollte auch klar sein warum durch
> 2 dividiert wird.
>  
> >  

> > Kann ich das so machen und jetzt so weiter vorgehen?:
>  >    [mm]\integral_{t0}^{t1}{\bruch{1}{m} \bruch{d\vec{p}^2}{dt} dt}[/mm]
>  
> >  

> > und [mm]\bruch{d\vec{p}^2}{dt}[/mm] ergit [mm]2(\vec{p}+\dot{\vec{p}})[/mm]
>  
> [notok]
>  Du solltest besser nochmal die Ableitungsregeln
> wiederholen ;-)
>  
> >  jetzt weiß ich nicht mehr weiter... es wird zwar durch 2

> > dividiert aber warum, ich versteh das nicht... oder hab ich
> > weiter oben schon einen fehler? danke nochmals
>  >  
>
> Gruß,
>  
> notinX

ok, danke. ja, mich hat das mit der produktregel in der vorlesung sehr verwirrt...
wenn ich jetzt deine Angabe löse:
[mm] \frac{1}{m}\frac{\mathrm{d}}{\mathrm{d}t}(\vec{p}\cdot\vec{p}) [/mm] =  [mm] \bruch{1}{m}2\vec{p} [/mm]  |:2
[mm] \bruch{1}{2m}\bruch{d\vec{p}^2}{dt} [/mm] = [mm] \bruch{\vec{p}}{m} [/mm] = v

hab ich es jetzt richtig verstanden? dann würde es so weitergehen:

= [mm] \integral_{t0}^{t1}{\bruch{1}{2m}\bruch{d\vec{p}^2}{dt} dt} [/mm]
jetzt kürze ich das dt und nehme wieder die ursprünglichen integrationsgrenzen:

= [mm] \integral_{\vec{p0}}^{\vec{p1}}{\bruch{d\vec{p}^2}{2m}} [/mm]
= [mm] \bruch{\vec{p_{1}}^2}{2m} [/mm] - [mm] \bruch{\vec{p_{0}}^2}{2m} [/mm]

hab ich es jetzt richtig? danke nomals für eure hilfe



Bezug
                                        
Bezug
Integration v. Arbeitsbeträgen: Antwort
Status: (Antwort) fertig Status 
Datum: 14:41 Fr 06.11.2015
Autor: notinX


> ok, danke. ja, mich hat das mit der produktregel in der
> vorlesung sehr verwirrt...
>  wenn ich jetzt deine Angabe löse:
>  
> [mm]\frac{1}{m}\frac{\mathrm{d}}{\mathrm{d}t}(\vec{p}\cdot\vec{p})[/mm]
> =  [mm]\bruch{1}{m}2\vec{p}[/mm]  |:2
>  [mm]\bruch{1}{2m}\bruch{d\vec{p}^2}{dt}[/mm] = [mm]\bruch{\vec{p}}{m}[/mm] =
> v

Nein, die Ableitung ist falsch. Wende die Kettenregel richtig an.

>  
> hab ich es jetzt richtig verstanden? dann würde es so
> weitergehen:
>  
> = [mm]\integral_{t0}^{t1}{\bruch{1}{2m}\bruch{d\vec{p}^2}{dt} dt}[/mm]
>  
> jetzt kürze ich das dt und nehme wieder die
> ursprünglichen integrationsgrenzen:

So einfach geht das nicht...

>  
> = [mm]\integral_{\vec{p0}}^{\vec{p1}}{\bruch{d\vec{p}^2}{2m}}[/mm]

Ein Integralzeichen ohne [mm] $\mathrm{d}t$ [/mm] am Ende macht keinen Sinn.

>  = [mm]\bruch{\vec{p_{1}}^2}{2m}[/mm] - [mm]\bruch{\vec{p_{0}}^2}{2m}[/mm]
>  
> hab ich es jetzt richtig? danke nomals für eure hilfe
>  
>  

Das Ergebnis stimmt, aber die Ableitungsregeln scheinen bei Dir noch nicht richtig zu sitzen.

Gruß,

notinX

Bezug
                                                
Bezug
Integration v. Arbeitsbeträgen: Frage (beantwortet)
Status: (Frage) beantwortet Status 
Datum: 17:29 Fr 06.11.2015
Autor: marc518205


> > ok, danke. ja, mich hat das mit der produktregel in der
> > vorlesung sehr verwirrt...
>  >  wenn ich jetzt deine Angabe löse:
>  >  
> >
> [mm]\frac{1}{m}\frac{\mathrm{d}}{\mathrm{d}t}(\vec{p}\cdot\vec{p})[/mm]
> > =  [mm]\bruch{1}{m}2\vec{p}[/mm]  
> Nein, die Ableitung ist falsch. Wende die Kettenregel
> richtig an.

ok, danke, aber ich blick jetz gar nicht mehr durch. was soll ich jetzt ableiten, wenn nicht [mm] \vec{p}^2? [/mm]
danke nochmals

Bezug
                                                        
Bezug
Integration v. Arbeitsbeträgen: Antwort
Status: (Antwort) fertig Status 
Datum: 18:04 Fr 06.11.2015
Autor: MathePower

Hallo marc518205,


> > > ok, danke. ja, mich hat das mit der produktregel in der
> > > vorlesung sehr verwirrt...
>  >  >  wenn ich jetzt deine Angabe löse:
>  >  >  
> > >
> >
> [mm]\frac{1}{m}\frac{\mathrm{d}}{\mathrm{d}t}(\vec{p}\cdot\vec{p})[/mm]
> > > =  [mm]\bruch{1}{m}2\vec{p}[/mm]  
> > Nein, die Ableitung ist falsch. Wende die Kettenregel
> > richtig an.
>  
> ok, danke, aber ich blick jetz gar nicht mehr durch. was
> soll ich jetzt ableiten, wenn nicht [mm]\vec{p}^2?[/mm]


Genau dieses sollst Du mit Hilfe der Kettenregel ableiten.

Mehr dazu: Kettenregel


>  danke nochmals


Gruss
MathePower

Bezug
                                                                
Bezug
Integration v. Arbeitsbeträgen: Frage (beantwortet)
Status: (Frage) beantwortet Status 
Datum: 19:51 Fr 06.11.2015
Autor: marc518205


> Hallo marc518205,
>  
>
> > > > ok, danke. ja, mich hat das mit der produktregel in der
> > > > vorlesung sehr verwirrt...
>  >  >  >  wenn ich jetzt deine Angabe löse:
>  >  >  >  
> > > >
> > >
> >
> [mm]\frac{1}{m}\frac{\mathrm{d}}{\mathrm{d}t}(\vec{p}\cdot\vec{p})[/mm]
> > > > =  [mm]\bruch{1}{m}2\vec{p}[/mm]  
> > > Nein, die Ableitung ist falsch. Wende die Kettenregel
> > > richtig an.
>  >  
> > ok, danke, aber ich blick jetz gar nicht mehr durch. was
> > soll ich jetzt ableiten, wenn nicht [mm]\vec{p}^2?[/mm]
>  
>
> Genau dieses sollst Du mit Hilfe der Kettenregel ableiten.
>  
> Mehr dazu:
> Kettenregel
>  
>
> >  danke nochmals

>
>
> Gruss
>  MathePower

ich denke das hauptproblem ist, dass ich nicht sehe wo ich hier die kettenregel anwenden soll. könnt ihr mir bitte sagen was hier zum innern und was zum außeren teil gehört? ich steh leider voll auf der leitung. danke für die geduld...


Bezug
                                                                        
Bezug
Integration v. Arbeitsbeträgen: Antwort
Status: (Antwort) fertig Status 
Datum: 20:17 Fr 06.11.2015
Autor: Event_Horizon

Hallo!

Es gilt zwar

[mm] \frac{d}{dx}x^2=2x [/mm]

Aber was ist mit

[mm] \frac{d}{dx}(f(x))^2 [/mm]  ?

Noch ein Hinweis: Die Einheiten verhalten sich beim Ableiten, als sei es einfache Bruchrechnung. Also

[mm] p^2 [/mm] ->   [mm] \frac{kg^2*m^2}{s^2} [/mm]

[mm] \frac{d}{dt}p^2 [/mm] ->   [mm] \frac{1}{s}*\frac{kg^2*m^2}{s^2}=\frac{kg^2*m^2}{s^3} [/mm]


Bezug
                                                                                
Bezug
Integration v. Arbeitsbeträgen: Frage (beantwortet)
Status: (Frage) beantwortet Status 
Datum: 08:35 Sa 07.11.2015
Autor: marc518205


> Hallo!
>  
> Es gilt zwar
>  
> [mm]\frac{d}{dx}x^2=2x[/mm]
>  
> Aber was ist mit
>
> [mm]\frac{d}{dx}(f(x))^2[/mm]  ?
>  
> Noch ein Hinweis: Die Einheiten verhalten sich beim
> Ableiten, als sei es einfache Bruchrechnung. Also
>  
> [mm]p^2[/mm] ->   [mm]\frac{kg^2*m^2}{s^2}[/mm]

>  
> [mm]\frac{d}{dt}p^2[/mm] ->  

> [mm]\frac{1}{s}*\frac{kg^2*m^2}{s^2}=\frac{kg^2*m^2}{s^3}[/mm]
>

herzlichen dank für eure hilfe. also ich versuchs noch mal...
ich nehm die kettenregel:

[mm] \frac{d}{dx}(f(x))^2 [/mm]  dann ist die potenz (danke notinX) die äußere funkton.
[mm] p^2 [/mm] --> 2p = 2mv [mm] =2\bruch{kgm}{s} [/mm]

jetz bilde ich die innere ableitung also von p = [mm] \bruch{kgm}{s} [/mm]
da es sich um einen bruch handelt wende ich die quotientenregel an. d. h. wenn [mm] (\bruch{f}{g})' [/mm] = [mm] \bruch{f'g-fg'}{g^2} [/mm] dann bedeutet das für mich:
[mm] (\bruch{kgm}{s})' [/mm] = [mm] \bruch{0kgm-kgm}{s^2} [/mm] = [mm] -\bruch{kgm}{s^2} [/mm]

jetzt wieder zurück zur kettenregel:
[mm] 2*\bruch{kgm}{s}*-\bruch{kgm}{s^2} [/mm] = [mm] -2\bruch{kg^2m^2}{s^3} [/mm]
ist es so nun richtig? irgendwie stört mich das minus...




Bezug
                                                                                        
Bezug
Integration v. Arbeitsbeträgen: Antwort
Status: (Antwort) fertig Status 
Datum: 12:18 Sa 07.11.2015
Autor: notinX


> > Hallo!
>  >  
> > Es gilt zwar
>  >  
> > [mm]\frac{d}{dx}x^2=2x[/mm]
>  >  
> > Aber was ist mit
> >
> > [mm]\frac{d}{dx}(f(x))^2[/mm]  ?
>  >  
> > Noch ein Hinweis: Die Einheiten verhalten sich beim
> > Ableiten, als sei es einfache Bruchrechnung. Also
>  >  
> > [mm]p^2[/mm] ->   [mm]\frac{kg^2*m^2}{s^2}[/mm]

>  >  
> > [mm]\frac{d}{dt}p^2[/mm] ->  

> > [mm]\frac{1}{s}*\frac{kg^2*m^2}{s^2}=\frac{kg^2*m^2}{s^3}[/mm]
>  >

>
> herzlichen dank für eure hilfe. also ich versuchs noch
> mal...
>  ich nehm die kettenregel:
>  
> [mm]\frac{d}{dx}(f(x))^2[/mm]  dann ist die potenz (danke notinX)
> die äußere funkton.
>  [mm]p^2[/mm] --> 2p = 2mv [mm]=2\bruch{kgm}{s}[/mm]

Das ist totaler Unfug. Du setzt den Impuls mit seiner Einheit gleich.
Ich glaube, Du hast den Hinweis mit den Einheiten von Event_Horizon total falsch verstanden. Also nochmal:
Nehmen wir den Ort $x(t)$ mit der Einheit [mm] $[x(t)]=\frac{m}{s}$. [/mm] Dann hat die Geschwindigkeit [mm] $v(t)=\dot{x}(t)=\frac{\mathrm{d}}{\mathrm{d}t}x(t)$ [/mm] als Ableitung des Ortes die Einheit [mm] $[v(t)]=\frac{m}{s^2}$, [/mm] weil man bei der Ableitung quasi nochmal 'durch t teilt'.

Du hast z.B. geschrieben:

> und $ [mm] \bruch{d\vec{p}^2}{dt} [/mm] $ ergit $ [mm] 2(\vec{p}+\dot{\vec{p}}) [/mm] $

Das kann schon allein deshalb nicht stimmen, weil Du hier zwei Terme mit unterschiedlichen Einheiten addierst.

Schreib mal die Ableitung [mm] $\frac{\mathrm{d}}{\mathrm{d}x}(f(x))^{2}$ [/mm] in vernünftiger Notation auf.

Wenn Du das geschafft hast, ersetze $f(x)$ durch $p(t)$ und die Ortsableitung durch eine Zeitableitung.

>  
> jetz bilde ich die innere ableitung also von p =
> [mm]\bruch{kgm}{s}[/mm]

[notok]

>  da es sich um einen bruch handelt wende ich die
> quotientenregel an. d. h. wenn [mm](\bruch{f}{g})'[/mm] =
> [mm]\bruch{f'g-fg'}{g^2}[/mm] dann bedeutet das für mich:
>  [mm](\bruch{kgm}{s})'[/mm] = [mm]\bruch{0kgm-kgm}{s^2}[/mm] =
> [mm]-\bruch{kgm}{s^2}[/mm]

Auch und gerade als Physiklehrer solltest Du Dir eine vernünftige, mathematisch saubere Notation angewöhnen. Zumindest so sauber, bis kein anderer Physiker mehr was zu meckern hat ;-)

Hättest Du das vernünftig notiert, wäre Dir hoffentlich aufgefallen, dass Du nacht t ableiten willst, in Deinem Term aber gar kein t vorkommt. (Ganz davon abgesehen, dass es Unfug ist eine Physikalische Größe mit ihrer Einheit gleichzusetzen.)

>  
> jetzt wieder zurück zur kettenregel:
>  [mm]2*\bruch{kgm}{s}*-\bruch{kgm}{s^2}[/mm] =
> [mm]-2\bruch{kg^2m^2}{s^3}[/mm]
>  ist es so nun richtig? irgendwie stört mich das minus...
>  

Mich stört da viel mehr als das minus. Korrigiere das lieber mal schnell...

Gruß,

notinX

Bezug
                                                                                                
Bezug
Integration v. Arbeitsbeträgen: Frage (beantwortet)
Status: (Frage) beantwortet Status 
Datum: 14:44 Sa 07.11.2015
Autor: marc518205

danke für die hinweise. ich gebs zu ich bin ein dummes rindvieh... ich versuchs nochmal ganz einfach, da ich vermutlich viel zu kopliziert gedacht habe...

ich möchte [mm] p^{2} [/mm] ableiten.
1.) mit der kettenregel:
dann erhalte ich 2*p*p'

2.) mit der produktregel:
dann erhalt ich auch 2*p*p'

da ich jetzt bei beiden varianten das selbe rausbekomme, dürfte es doch stimmen oder?

dann wäre mir auch halbwegs klar warum das ganze dann durch 2 dividiert wird und folgendes rauskommt:

[mm] \integral_{t0}^{t1}{\bruch{1}{2m} \bruch{d\vec{p}^2}{dt} dt} [/mm]

bin ich jetzt auf dem richtigen weg?




Bezug
                                                                                                        
Bezug
Integration v. Arbeitsbeträgen: Antwort
Status: (Antwort) fertig Status 
Datum: 14:56 Sa 07.11.2015
Autor: notinX


> danke für die hinweise. ich gebs zu ich bin ein dummes
> rindvieh... ich versuchs nochmal ganz einfach, da ich
> vermutlich viel zu kopliziert gedacht habe...
>  
> ich möchte [mm]p^{2}[/mm] ableiten.
>  1.) mit der kettenregel:
>  dann erhalte ich 2*p*p'

[ok]

>  
> 2.) mit der produktregel:
>  dann erhalt ich auch 2*p*p'

[ok]

>  
> da ich jetzt bei beiden varianten das selbe rausbekomme,
> dürfte es doch stimmen oder?

Ja, so stimmts.

>  
> dann wäre mir auch halbwegs klar warum das ganze dann
> durch 2 dividiert wird und folgendes rauskommt:
>  
> [mm]\integral_{t0}^{t1}{\bruch{1}{2m} \bruch{d\vec{p}^2}{dt} dt}[/mm]
>  
> bin ich jetzt auf dem richtigen weg?
>  
>
>  

Ja, sieht gut aus. Jetzt noch den Hauptsatz der Differential- und Integralrechnung anwenden, dann bist Du fertig.

Gruß,

notinX

Bezug
                                                                                                                
Bezug
Integration v. Arbeitsbeträgen: Mitteilung
Status: (Mitteilung) Reaktion unnötig Status 
Datum: 15:01 Sa 07.11.2015
Autor: marc518205

herzlichen dank für eure hilfe und vor allem für eure geduld, ich hatte voll das brett vorm kopf... danke

Bezug
                                                                                        
Bezug
Integration v. Arbeitsbeträgen: Antwort
Status: (Antwort) fertig Status 
Datum: 14:46 Sa 07.11.2015
Autor: Event_Horizon

Hallo!

Ich weiß nicht, wo genau es bei dir beim Ableiten hakt. Aber ich denke, es bringt dir und uns nichts, da jetzt noch länger drauf rum zu hacken.

Daher:

Frei nach dem Motto "innere mal äußere" ergibt sich hier:


[mm] \frac{d}{dt}\blue{\vec{p}(t)}^\red{2}=\blue{\dot{\vec{p}}}*\red{2\vec{p}(t)} [/mm]

Und das mit den Einheiten hast du auch irgendwie fürchterlich mißverstanden, dazu hat notinX schon was geschrieben. Auch das schreib ich mal hierfür auf:

[mm] \frac{1}{s}*\red{\left( \blue{\frac{kg*m}{s}} \right)^2}=\blue{\frac{kg*m}{s^2}}*\red{\frac{kg*m}{s}} [/mm]

Bei genauerer Betrachtung siehst du, daß links und rechts die gleiche Einheit steht, ganz so, wie es sein muß.

Bezug
                                                                        
Bezug
Integration v. Arbeitsbeträgen: Antwort
Status: (Antwort) fertig Status 
Datum: 21:53 Fr 06.11.2015
Autor: notinX


> ich denke das hauptproblem ist, dass ich nicht sehe wo ich
> hier die kettenregel anwenden soll. könnt ihr mir bitte
> sagen was hier zum innern und was zum außeren teil
> gehört? ich steh leider voll auf der leitung. danke für
> die geduld...
>  

Die äußere Funktion ist in dem Fall die Potenz ($^2$) und die innere ist $p(t)$. Du kannst es aber genausogut mit der Produktregel berechnen, nämlich die Ableitung des Produktes aus [mm] $\vec [/mm] p$ mit sich selbst.

Gruß,

notinX

Bezug
Ansicht: [ geschachtelt ] | ^ Forum "HochschulPhysik"  | ^^ Alle Foren  | ^ Forenbaum  | Materialien


^ Seitenanfang ^
www.matheraum.de
[ Startseite | Forum | Wissen | Kurse | Mitglieder | Team | Impressum ]